From b449fe20d387a3ae010cd744a7a1ceb9b006241d Mon Sep 17 00:00:00 2001 From: William Trevor King Date: Fri, 7 Aug 2009 09:29:03 -0400 Subject: [PATCH] Fixed typo in Young_and_Freedman_12/problem26.91.tex --- latex/problems/Young_and_Freedman_12/problem26.91.tex | 2 +- 1 file changed, 1 insertion(+), 1 deletion(-) diff --git a/latex/problems/Young_and_Freedman_12/problem26.91.tex b/latex/problems/Young_and_Freedman_12/problem26.91.tex index 5e74a0f..157e691 100644 --- a/latex/problems/Young_and_Freedman_12/problem26.91.tex +++ b/latex/problems/Young_and_Freedman_12/problem26.91.tex @@ -5,7 +5,7 @@ resistance $R_T$ of the infinite network is equal to \begin{equation} R_T = R_1 + \sqrt{R_1^2 + 2R_1R_2} \end{equation} -(\emph{Hint:} Since the network is infinite, the sestance of the +(\emph{Hint:} Since the network is infinite, the resistance of the network to the right of points $c$ and $d$ is also equal to $R_T$.) \begin{center} \begin{verbatim} -- 2.26.2